Help with what I think it's an impossible integral

Click For Summary
The discussion revolves around finding the surface area of a volume generated by rotating the equation 3y² = x(1-x)² around both axes. The user rewrites the equation for rotation around the x-axis and attempts to apply the surface area formula, but encounters difficulties with the integration process. There are concerns about the accuracy of the derivatives and the integrand used in the calculations, leading to confusion about the integration steps. The need for clearer notation, specifically using LaTeX for the integrals, is emphasized to improve readability and understanding. Overall, the conversation highlights the challenges faced in solving complex integrals in calculus.
stonecoldgen
Messages
108
Reaction score
0
So they give me the equation 3y2=x(1-x)2

The idea is to find the surface area of the volume obtained by rotating around both axes.



So let's start with a rotation around the x-axis, I decided to rewrite the equation as:
y=√(x(1-x2)/3)


I know that the surface area for a parallel rotation is S=∫2∏r√((dr/dx)2+1)

and I know the derivative of the function, so I end up with:



S=∫2∏r√2(1-3x2)/(6√(x-x3)/3))+1)

How the HELL am I supposed to integrate that?
 
Physics news on Phys.org
I think you made a mistake in determining the integrand. What you have for the dy/dx is not what I got when I differentiated the function (you also write the function as containing x(1-x)^2 initially and then when you rewrite it in terms of y you have an x(1-x^2) but I believe I got something different when I used either). Also, it doesn't look like you squared the dy/dx in the integrand. Finally, keep in mind that the r is equal to the value of the function, so you have to plug that in (You give the function in terms of y, but then you have r and dr/dx in the formula - those should be y and dy/dx... in this case, the radius is the height of the function)
 
stonecoldgen said:
I know that the surface area for a parallel rotation is S=∫2∏r√((dr/dx)2+1)

and I know the derivative of the function, so I end up with:

S=∫2∏r√2(1-3x2)/(6√(x-x3)/3))+1)

How the HELL am I supposed to integrate that?

I can't even read that. Please consider typing up those integrals in LaTeX.
 
Question: A clock's minute hand has length 4 and its hour hand has length 3. What is the distance between the tips at the moment when it is increasing most rapidly?(Putnam Exam Question) Answer: Making assumption that both the hands moves at constant angular velocities, the answer is ## \sqrt{7} .## But don't you think this assumption is somewhat doubtful and wrong?

Similar threads

  • · Replies 7 ·
Replies
7
Views
2K
  • · Replies 2 ·
Replies
2
Views
2K
  • · Replies 19 ·
Replies
19
Views
2K
Replies
3
Views
2K
  • · Replies 14 ·
Replies
14
Views
2K
  • · Replies 3 ·
Replies
3
Views
2K
Replies
2
Views
1K
  • · Replies 105 ·
4
Replies
105
Views
6K
  • · Replies 9 ·
Replies
9
Views
2K
  • · Replies 9 ·
Replies
9
Views
2K